Đến nội dung

Stranger411

Stranger411

Đăng ký: 24-04-2012
Offline Đăng nhập: 15-09-2015 - 12:18
***--

#347133 Tìm số nguyên $n> 1$ sao cho $\frac{2^{n...

Gửi bởi Stranger411 trong 16-08-2012 - 10:06

Bài trên lâu lắm rồi :D Mình mở rộng bằng căn nguyên thủy tí nữa cho nó mạnh :D

Mở rộng: Tìm tất cả các số nguyên $n>1$ sao cho tồn tại duy nhất số nguyên $a$ với $0 < a < n!$ sao cho:
\[n!|{a^n} + 1\]

BÀI TOÁN: Xác định tất cả các số nguyên $n> 1$ sao cho $\frac{2^{n}+1}{n^{2}}$ là một số nguyên.

Bài này còn 2 cách giải nữa bằng căn nguyên thủy và LTE ;)


#347129 Tìm số nguyên $n> 1$ sao cho $\frac{2^{n...

Gửi bởi Stranger411 trong 16-08-2012 - 09:52

BÀI TOÁN: Xác định tất cả các số nguyên $n> 1$ sao cho $\frac{2^{n}+1}{n^{2}}$ là một số nguyên.

Bài này ko cần phải dùng đến cấp của 1 số đâu :)

Bổ đề 1: Cho các số nguyên $m,n$ và $a>1$. Ta có: $\gcd \left( {{a^m} - 1,{a^n} - 1} \right) = {a^{\gcd \left( {m,n} \right)}} - 1$
Bổ đề 2: Nếu ${3^b}|{2^a} - 1 \Rightarrow {3^{b - 1}}|a$

Lời giải bài toán:
+ Khi $n=1$, bài toán thỏa mãn.
+ Khi $n>1 \Rightarrow n$ lẻ.
Gọi $p$ là ước nguyên tố lẻ nhỏ nhất của $n$ nên $\gcd \left( {p - 1,n} \right) = 1$.
Ta có: $p|{2^n} + 1|{2^{2n}} - 1$
Theo định lí Ferma nhỏ, ta có: $p|{2^{p - 1}} - 1$.
Áp dụng bổ đề 1, ta được: $p|\gcd \left( {{2^{p - 1}} - {{1,2}^{2n}} - 1} \right) = {2^{\gcd \left( {2n,p - 1} \right)}} - 1$
mà $\gcd \left( {2n,p - 1} \right) \leqslant 2 \Rightarrow p|3 \Rightarrow p = 3$
Đặt $n = {3^k}d$. Dùng bổ đề 2, ta có: ${3^{2k}}|{n^2}|{2^{2n}} - 1 \Rightarrow {3^{2k - 1}}|n \Rightarrow k = 1$
(*) Nếu $d>1$. Gọi $q$ là ước nguyên tố nhỏ nhất của $d$ nên $q \ge 5$.
Lập luận tương tự như trên, ta có: $q=7$
Vậy nên $7|n|{2^n} + 1$. Điều này vô lí vì ${2^n} + 1 \equiv 2,3,5(\bmod 7)$.
(*) Nếu $d=1$, ta có: $n=3$.
Vậy $n=1$ và $n=3$ thỏa mãn đề bài.


#346650 ${p_{k + 1}}|{p_1} + {p_2} +...

Gửi bởi Stranger411 trong 14-08-2012 - 11:48

Cho số nguyên $n \ge 2$.
Chứng minh rằng tồn tại 1 bộ hoán vị ${p_1},{p_2}, \ldots ,{p_n}$ của $1,2, \ldots ,n$ sao cho ${p_{k + 1}}|{p_1} + {p_2} + \ldots + {p_k}$ với $k = 1,2, \ldots ,n-1$


#346612 $\sum\limits_{k = 1}^{p - 1} {\l...

Gửi bởi Stranger411 trong 13-08-2012 - 23:32

Cho p là số nguyên tố .CMR $\sum\limits_{k = 1}^{p - 1} {\left\lfloor {\frac{{{k^3}}}{p}} \right\rfloor } = \frac{{(p + 1)(p - 1)(p - 2)}}{4}$

Khuya rồi !! Lên Vmf chém thử vài bài cho vui :D

Với $1 \leqslant k \leqslant p - 1$thì $\left\{ \begin{gathered}
{k^3} \equiv 0(\bmod p) \\
{\left( {p - k} \right)^3} \equiv - {\left( k \right)^3} \\
\end{gathered} \right.$
Nên $\left( {\frac{{{k^3}}}{p} - \left\lfloor {\frac{{{k^3}}}{p}} \right\rfloor } \right) + \left( {\frac{{{{\left( {p - k} \right)}^3}}}{p} - \left\lfloor {\frac{{{{\left( {p - k} \right)}^3}}}{p}} \right\rfloor } \right) = 1$.
Từ đó, ta được:
$\begin{gathered}
\sum\limits_{k = 1}^{p - 1} {\left\lfloor {\frac{{{k^3}}}{p}} \right\rfloor } = \sum\limits_{k = 1}^{p - 1} {\frac{{{k^3}}}{p}} - \sum\limits_{k = 1}^{p - 1} {\left( {\frac{{{k^3}}}{p} - \left\lfloor {\frac{{{k^3}}}{p}} \right\rfloor } \right)} \\
= \frac{1}{p}\sum\limits_{k = 1}^{p - 1} {{k^3}} - \frac{1}{2}\sum\limits_{k = 1}^{p - 1} {\left( {\frac{{{k^3}}}{p} - \left\lfloor {\frac{{{k^3}}}{p}} \right\rfloor } \right)} + \left( {\frac{{{{\left( {p - k} \right)}^3}}}{p} - \left\lfloor {\frac{{{{\left( {p - k} \right)}^3}}}{p}} \right\rfloor } \right) \\
= \frac{1}{p}{\left( {\frac{{p\left( {p - 1} \right)}}{2}} \right)^2} - \frac{{p - 1}}{2} = \frac{{\left( {p + 1} \right)\left( {p - 1} \right)\left( {p - 2} \right)}}{4} \\
\end{gathered}$

ps Chú Nguyenta98 post trước mình 4 phút.


#346443 Các bài toán về số Ferma - Các bài giảng của LEUNG Tat-Wing

Gửi bởi Stranger411 trong 13-08-2012 - 13:46

Mới tìm được trong đống bài giảng của ông LEUNG Tat-Wing 1 bài viết rất hay về Số Ferma.

Công nhận mấy giáo sư người Tàu rất đáng khâm phục. Mời các bạn tham khảo nó :D

File gửi kèm




#346426 Tìm các số nguyên dương $a,b,c$ sao cho $\frac{a^{2}+b^{2...

Gửi bởi Stranger411 trong 13-08-2012 - 12:03

ý m` là không có cách của bạn đâu,nếu 2a+b chia hết cho p thì làm quoái j` mà vô lí chứ,cũng giống như a mũ 2 + b mũ 2 có ước nguyên tố dạng thì 4p+3 thì a,b cùng chia hết cho SNT đó,làm j` có chuyện vô lí ở đâu,khi sử dụng kí hiêu lengdre ta đã ngầm hiểu tử số của nó không chia hết cho mẫu r` :lol:

Trước khi nói cái gì thì nên coi lại kiến thức của mình một tí đi nhá ;)

Định nghĩa về kí hiệu Lengdre:
Cho số nguyên tố $p$ và số nguyên $a$. Khi đó, ta có:
+$\left( {\frac{{ a}}{p}} \right) = 1$ nếu $a \not \vdots p$ và $a$ là số chính phương $mod(p)$
+$\left( {\frac{{ a}}{p}} \right) = -1$ nếu $a \not \vdots p$ và $a$ không là số chính phương $mod(p)$
+$\left( {\frac{{ a}}{p}} \right) = 0$ nếu $a \vdots p$

Trờ lại bài toán:
Vì vậy nếu $\left( {\frac{{ - 3}}{p}} \right) = 1$ thì hoàn toàn vô lí vì ta chọn $p \equiv 2(\bmod 3)$


#346020 Tìm các số nguyên dương $a,b,c$ sao cho $\frac{a^{2}+b^{2...

Gửi bởi Stranger411 trong 11-08-2012 - 23:47

He he áp dụng cái bổ đề anh Tường nói thì bài này làm ngon
Giải như sau:
Bổ đề: $p \in \mathbb{P}, p \equiv 2 \pmod{3}, a^2+3b^2 \vdots p \Leftrightarrow p|a,b$

Em Tạ giải kinh quá :P
Chắc thằng Tường nó chả bao giờ theo kịp đâu :P

Cách khác:
Đẳng thức được viết lại như sau $(a+b+c)^2=(3k+2)(ab+bc+ca)$
Chọn số nguyên tố $p$ sao cho $\left\{ \begin{gathered}
{p^{2a - 1}}|3k + 2 \\
{p^{2a}}|3k + 2 \\
\end{gathered} \right.$
$ \Rightarrow \left\{ \begin{gathered}
{p^a}|a + b + c \\
p|ab + bc + ca \\
\end{gathered} \right.$
$c \equiv - a - b(\bmod p) \Rightarrow p|a^2 + ab + b^2 \Rightarrow p|{(2a + b)^2} + 3{b^2}$
$ \Rightarrow \left( {\frac{{ - 3}}{p}} \right) = 1$. Và điều này vô lí vì $p \equiv 2(\bmod 3)$.
Vậy không tồn tại $a,b,c$ thỏa mãn bài toán. $\blacksquare$


#345861 Ảnh thành viên

Gửi bởi Stranger411 trong 11-08-2012 - 14:58

Hình đã gửi
Up cái này phát!
Ông quả nhìn to quá!

Mấy ông ơi !!
cái bảng đó bây h ai giữ thế ;p
Bữa nào bán đấu giá trên Vmf đi :D


#345840 $\varphi (5^{m}-1)=5^{n}-1$

Gửi bởi Stranger411 trong 11-08-2012 - 13:36

giả sử m,n là các số nguyên dương sao cho $\varphi (5^{m}-1)=5^{n}-1$.cmr UCLN của m và n lớn hơn 1

Very beautiful !! A problem in quadratic residue ;)

Bổ đề: Cho các số nguyên $m,n$ và $a>1$. Ta có: $\gcd \left( {{a^m} - 1,{a^n} - 1} \right) = {a^{\gcd \left( {m,n} \right)}} - 1$

Lời giải bài toán:
Giả sử $gcd(m,n)=1$
Ta xét phân tích cơ sở: ${5^m} - 1 = {2^a}{p_1}^{{a_1}} \ldots {p_k}^{{a_k}}$ với ${p_i} > 2$
Vậy nên:
\[{5^n} - 1 = \varphi \left( {{5^m} - 1} \right) = {2^{a - 1}}{p_1}^{{a_1} - 1}...{p_k}^{{a_k} - 1}\prod\limits_{i = 1}^k {\left( {{p_i} - 1} \right)} \]
Vì ${2^a}|{5^n} - 1$. Kết hợp với bổ đề, ta có: $\gcd \left( {{5^m} - {{1,5}^n} - 1} \right) = 5 - 1 = 4$
Vậy nên $a=2$
Vì ${2^3}|{5^m} - 1 \Rightarrow m = 2k + 1 \Rightarrow {p_i}|5.{\left( {{5^k}} \right)^2} - 1$
Vậy 5 là số chính phương (mod $p_i$) nên ${p_i} \equiv - 1(\bmod 5)$
$$ \Rightarrow \left\{ \begin{gathered}
{\left( { - 1} \right)^k} = 1 \\
{\left( { - 2} \right)^{k + 1}} \equiv 1(\bmod 5) \\
\end{gathered} \right.
\Rightarrow \left\{ \begin{gathered}
k \equiv 0(\bmod 2) \\
k \equiv 3(\bmod 4) \\
\end{gathered} \right.$$
Mâu thuẫn,
Vậy $gcd(m,n)>1$.


#344948 CMR$\left( {\frac{{p - 1}}{2...

Gửi bởi Stranger411 trong 08-08-2012 - 23:38

Bài toán trên còn 1 cách phát biểu khác như sau:
Hình đã gửi


#343961 $A = \frac{{{a^2} + {b^2} + {c^2...

Gửi bởi Stranger411 trong 06-08-2012 - 12:19

Tìm các số nguyên dương a,b,c đề $A = \frac{{{a^2} + {b^2} + {c^2}}}{{abc}} \in {N^*}$

Bài này vô số nghiệm.

Bước 1: Ta cần chứng minh: $A=1$ hoặc $A=3$.

Bước 2: Ta tìm $a,b,c$
Với $A=3$, nghiệm của phương trình $a^2+b^2+c^2=3abc$ là
\[\left\{ \begin{gathered}
z = 1 \\
x = {u_n} \\
y = {u_{n + 1}} \\
\end{gathered} \right.\]
Với
\[\left\{ \begin{gathered}
{u_0} = 1,{u_1} = 1\\
{u_{n + 1}} = 3{u_n} - {u_{n - 1}}\\
\end{gathered} \right.\]

Với $A=1$. Đặt $a=3x,b=3y,c=3z$, ta được phương trình trên.
Dẫn đến một bộ nghiệm tương tự ;)


#343075 MOSP 2001 by Cecil Rousseseau

Gửi bởi Stranger411 trong 03-08-2012 - 13:15

Problem: $a_n$ kí hiệu là số tập con không rỗng của $S$ thỏa mãn rằng:
(i) $S\subseteq${$1$, $2$, $...$, $n$};
(ii) tất cả các phần tử của $S$ đều cung tính chẵn, lẻ.
(iii) mỗi phân tử $k\in{S}$ thỏa mãn $k\geq2|S|$.
Tìm công thức tường minh cho $a_n$

Bài này lâu rồi, sử dụng phép chia nhóm là được :)

Ta có: $ a_{2m-1}= 2(F_{m+1}-1) $ và $ a_{2m}= F_{m+3}-2 $
với $m\ge1$ và $F_{m}$ là số Fibonacci thứ $m$.


Lời giải:
Đặt $T_{n}=\{S\in\{1,2,\cdots,n\}\}$ thỏa $(ii)$ và $(iii)$
Chia $ T_{n+4} $ thành 3 tập con:

Phần 1: $A_{n+4}=\{S\in T_{n+4}\ ;\ 1,2\notin S,\ \forall k\in S, k\geq 2|S|+2\}$
Xây dựng $ f\ :\ \mathcal{P}(\{1,2,\cdots,n+2\})\rightarrow\mathcal{P}(\{1,2,\cdots,n+4\}) $ thỏa:
$$f(\{x_{1},x_{2},\cdots,x_{k}\}) =\{x_{1}+2,x_{2}+2,\cdots,x_{k}+2\}$$
Ta được: $ f(T_{n+2}) = A_{n+4} $ nên $ |A_{n+4}|=|T_{n+2}|$

Phần 2: $ B_{n+4}=\{S\in T_{n+4}\ ;\ 1,2\notin S,\ \exists k\in S, k < 2|S|+2\} $
Tương tự, ta được:

$f(\phi) =\{3\}$
$f\left( {\left\{ {{x_1},{x_2},...,{x_k}} \right\}} \right) = \left\{ {{x_1} + 4,{x_2} + 4,...,{x_k} + 4,2k} \right\}$
nếu các phần tử cùng chẳn.
$f\left( {\left\{ {{x_1},{x_2},...,{x_k}} \right\}} \right) = \left\{ {{x_1} + 4,{x_2} + 4,...,{x_k} + 4,2k+1} \right\}$
nếu các phần tử cùng lẽ.
Suy ra: $|B_{n+4}|=|T_{n}|+1 $

Phần 3: $ C_{n+4}=\{ S\in T_{n+4}\ ;\ 1\in S\ \mathrm{or}\ 2\in S\} $
Tương tự, ta có: $ |T_{n+4}|=|T_{n+2}|+|T_{n}|+2 $

Từ đó, ta chứng minh được: $ a_{2m-1}= 2(F_{m+1}-1) $ và $ a_{2m}= F_{m+3}-2 $



#343028 có tất cả bao nhiêu số tự nhiên n<m sao cho m l n(2n+1)(5n+2)

Gửi bởi Stranger411 trong 03-08-2012 - 11:27

cho m=20072008 ,hỏi có tất cả bao nhiêu số tự nhiên n<m sao cho m l n(2n+1)(5n+2)

\[{\text{m}} = {\text{2}}{007^{2008}} = {3^{4016}}{223^{2008}}\]
Ta có: $m|n\left( {2n + 1} \right)\left( {5n + 2} \right)$
$ \Rightarrow m|10n\left( {10n + 5} \right)\left( {10n + 4} \right)$ $(1)$

Đặt $10n=x$, ta được: $m|x\left( {x + 5} \right)\left( {x + 4} \right)$

Đặt ${q_1} = {3^{4016}},{q_2} = {223^{2008}}$. Vì $\gcd \left( {{q_1},{q_2}} \right) = 1$

Nên \[{\text{(1)}} \Leftrightarrow \left\{ \begin{gathered}
x\left( {x + 5} \right)\left( {x + 4} \right) \equiv 0(\bmod {q_1}) \\
x\left( {x + 5} \right)\left( {x + 4} \right) \equiv 0(\bmod {q_2}) \\
\end{gathered} \right.\]
Mà $x \equiv 0(\bmod 10)$

Nên $x$ là nghiệm của hệ:
\[\left\{ \begin{gathered}
x \equiv {r_1}(\bmod {q_1}) \\
x \equiv {r_2}(\bmod {q_2}) \\
x \equiv 0(\bmod 10) \\
\end{gathered} \right.\]
với ${r_1},{r_2} \in \left\{ {0; - 4; - 5} \right\}$

Theo định lí Thặng Dư Trung Hoa, hệ có 1 nghiệm $(\bmod 10{q_1}{q_2})$
với mỗi cặp ${r_1},{r_2}$ chỉ tồn tại 1 nghiệm x.

Có tất cả $3^2$ cách chọn ${r_1},{r_2}$ nên có 9 số $x$ thỏa.
Suy ra có 9 số $n$ thỏa mãn bài toán.


#340310 Chứng minh: $a = {10^k}$

Gửi bởi Stranger411 trong 26-07-2012 - 01:01

Cho số nguyên dương $a$ thỏa mãn $S\left( {{a^n} + n} \right) = 1 + S\left( n \right)$ với mọi số tự nhiên $n$ lớn tùy ý.
Chứng minh $a$ là một lũy thừa của $10$.

- Kvant -




#340308 $S\left( {{a_n}} \right)$ không chia...

Gửi bởi Stranger411 trong 26-07-2012 - 00:51

Cho 1 dãy số nguyên $\left( {{a_n}} \right)$ phân biệt thỏa mãn ${{a_n} \leqslant 4999n}$ ${\forall n \geqslant 1}$.
Chứng minh có vô hạn số $n$ sao cho $S\left( {{a_n}} \right) $ không chia hết cho $5$